User Avatar
istvanvanvianen330
Joined
Apr 2025
Subscription
Free
User Avatar
istvanvanvianen330
Thursday, Apr 30 2020

Thank you for your input!

Just to clarify, I am not disputing the fact that the gap must be filled. My problem with the answer choice is that the scope of a necessary assumption should precisely meet only ‘generally’ (after all that is necessary for the stimulus to work) and not an absolute conditional relationship (a stronger claim with a higher burden of proof, hence it can’t be necessary). Sure, the sufficient assumption would make it valid, but it is not necessary to do so. (in this light, I suppose I am saying exactly the reverse of your last two sentences).

If we negate answer choice E, we get something like “Humans could make wiser choices even if emotional tendencies haven’t changed.” If this was a correct answer choice, then it should completely wreck the argument. But, right now ‘generally’ still enables that some people indeed could make wiser choices even if emotional blah blah haven’t changed. If this ‘some’ relationship holds, we simply can’t conclude that there is a conditional relationship between the two. For all we know, it is not even applicable to 49% of all people (generally~~most).

Therefore, a NA answer choice should be something like this “Usually wise decisions at least in part require a change in emotional tendencies.” This fills in the gap in the most bare bone, necessary way without making a claim that is 1) invalid and 2) more exclusive than what the argument actually needs.

PrepTests ·
PT149.S4.Q7
User Avatar
istvanvanvianen330
Thursday, Apr 30 2020

-_- <----- Summarizes my feelings about this question perfectly

User Avatar

Thursday, Apr 30 2020

istvanvanvianen330

PT 72 S2 Q12 NA Question

I am confused about the word “generally” in the conclusion.

To Recap The argument form in Lawgic:

P1: Emotional Tendencies /(Changed)

Required Premise: Emotional Tendencies /(Changed)-> Generally /(Able to choose more wisely)

Conclusion: Generally /(Able to choose more wisely)

Answer choice E seems to bridge this quite well, (Able to choose more wisely)->Emotional Tendencies (Changed)

However, the conclusion is qualified with the word “generally,” which implies that that it holds true “in most cases.”

This, however, implies that SOME people can choose more wisely even if emotional Tendencies are NOT changed. This is precisely the negation of the conditional relationship between 'Emotional Tendencies' and 'Choosing more wisely.'

If all of the above is correct, then the sufficient assumption stipulated by answer choice E is presumably not even valid in all cases. So how can it possibly be a necessary assumption as well?

If the necessary assumption’s role is to put the argument on life support (to use JY’s phrase), then I presume the correct answer should read something like this:

“Usually, wise decisions at least in part require a change in emotional tendencies.”

Any thoughts on this would be greatly appreciated.

PrepTests ·
PT148.S3.Q12
User Avatar
istvanvanvianen330
Thursday, Apr 30 2020

#help

I am confused about the word “generally” in the conclusion.

Recap The argument form in Lawgic:

P1: Emotional Tendencies /(Changed)

Required Pemise: Emotional Tendencies /(Changed)-> Generally /Able to choose more wisely

Conclusion: Generally /(Able to choose more wisely)

However, the conclusion is qualified with the word “generally,” which implies that that it holds true “in most cases.”

This, however, implies that SOME people can choose more wisely even if emotional Tendencies are NOT changed. This is precisely the negation of the conditional relationship between Emotional Tendencies and Choosing more wisely.

If all of the above is correct, then the sufficient assumption stipulated by answer choice E is presumably not even valid. In this case, how can it possibly be a necessary assumption as well?

If the necessary assumption’s role is to put the argument on life support (to use JY’s phrase), then I presume the correct answer should read something like this:

“Wise decisions at least in part require a change in emotional tendencies.”

Any thoughts on this would be greatly appreciated.

PrepTests ·
PT126.S4.Q19
User Avatar
istvanvanvianen330
Tuesday, Mar 24 2020

I was initially hesitant to lock In C, even after mapping out the Lawgic correctly, due to fact that it requires the assumption that ‘one must live in or be member of a society that defends freedom of thought in order to have freedom of thought.” But why does this have to be true? One could easily imagine a situation where an isolated erudite goes to the forest to freely think with the explicit purpose of avoiding society (In fact there are tangible examples for this in the real world, rendering this not one of those “common sense” LSAT assumptions). Furthermore, it could be entirely possible that the aforementioned erudite, who has freedom of thought, furthermore, possesses intellectual discipline. This scenario of course would render the professors argument invalid.

In short, this means that the argument is NOT 100% airtight and thus not meeting the criteria for validity. This makes it quite confusing for me since this is a sufficient assumption question where the burden of proof for validity is a necessary condition for the answer choice.. Could somebody explain this?

#HELP

PrepTests ·
PT137.S2.Q13
User Avatar
istvanvanvianen330
Thursday, Apr 23 2020

Premise 1: Most builders do NOT consider Papercrete to be promising for LARGE scale construction.

Premise 2: SMALL scale builders, who regularly build with Papercrete, think that it is promising.

Premise 3: Builders who regularly use a material are familiar with the properties.

Conclusion: It is likely that Papercrete is promising for LARGE scale construction.

Potential Gap:

1) The argument attempts to imply that the builders, who do not consider the material promising, are actually NOT familiar. But why does this have to be the case?

To put this into Lawgic:

Promising (regular users)-> Familiar

/Promising (most builders for large scale)

From this, the argument concludes, that /Promising must imply /Familiar (in order to draw the contrapositive justification)

But this is not a valid, when the sufficient is denied, the necessary condition may or may not be met. The builders of the large scale project may very well be familiar.

A) I don’t believe that this is descriptively accurate. The argument does not conflate what is promising for small scale construction to be promising for large scale. Regardless, this is certainly not the flaw. Instead, it seems to inappropriately draw an inference about familiarity on large scale construction.

B) Descriptively inaccurate. Seems the opposite?

C) Certainly descriptively inaccurate.

E Descriptively accurate but it is not the flaw. We might as well consider cats.

F CORRECT, closest match to prehpase

PrepTests ·
PT155.S4.Q21
User Avatar
istvanvanvianen330
Sunday, May 17 2020

Never thought i'd use an equation for the LSAT but here we are. Oddly, this was the fastest way for me on the timed run:

.38(Pulham and Westerville)=.29(Westerville) + X(Pulham)

.38(P) + .38(W) = .29(W )+ X(P)

.38(P) + .9(W) = X(P)

.38 + .9(W/P) =X

Proves answer choice A, because no matter the size of W and P, the result will be greater than .38%.

LSAT writers keep coming up with new stuff, lesson learned XD

PrepTests ·
PT151.S4.Q13
User Avatar
istvanvanvianen330
Thursday, May 07 2020

"piling garbage on top of garbage" LOL

PrepTests ·
PT102.S2.Q21
User Avatar
istvanvanvianen330
Monday, Apr 06 2020

Cookie cutter Lawgic:

/(POP) faculty -> modify

/(POP) students -> adopt new

From the inclusive OR statement:

(Pop) faculty -> /(POP) students -> adopt new

(POP) faculty -> adopt new

(POP) students -> /(POP) faculty -> modify

(POP) students -> modify

E: is correct as it exactly matches our inference

A: cannot refer this, nobody stated what we should do to influence popularity amongst students and faculty, just what should be done if the parking policy happens to be factually unpopular with either faculty or students.

B: modify is A necessary condition, can’t make an inference with modify in sufficient (at best /modify)

C: we should only modify, if it is either unpopular with faculty or popular with students. Not reducing popularity with the faculty does not match any of these sufficiency triggers.

D: invalid, answer should trigger ‘modify’ instead of ‘adopting new.‘

PrepTests ·
PT148.S1.Q8
User Avatar
istvanvanvianen330
Saturday, May 02 2020

JY was surprisingly nice to answer choice B in this one lol

Confirm action

Are you sure?